0% found this document useful (0 votes)
217 views18 pages

Analisis Funcional

The document contains 11 problems from the course AMAT 617. The problems involve finding maximal elements of sets with respect to divisibility, showing properties of subadditive functionals on normed spaces, properties of bounded linear operators and their adjoints, annihilators of subspaces, and concepts related to meager and comeager sets in complete metric spaces. Solutions to each problem are provided in detail.

Uploaded by

ricky201201
Copyright
© © All Rights Reserved
We take content rights seriously. If you suspect this is your content, claim it here.
Available Formats
Download as PDF, TXT or read online on Scribd
0% found this document useful (0 votes)
217 views18 pages

Analisis Funcional

The document contains 11 problems from the course AMAT 617. The problems involve finding maximal elements of sets with respect to divisibility, showing properties of subadditive functionals on normed spaces, properties of bounded linear operators and their adjoints, annihilators of subspaces, and concepts related to meager and comeager sets in complete metric spaces. Solutions to each problem are provided in detail.

Uploaded by

ricky201201
Copyright
© © All Rights Reserved
We take content rights seriously. If you suspect this is your content, claim it here.
Available Formats
Download as PDF, TXT or read online on Scribd
You are on page 1/ 18

Assignment 3

AMAT 617

Mark Girard
7 March 2014

Problem 1 (Problem 4, Chapter 4.1, p. 212). Find all maximal elements of M with respect to the partial
ordering m ≤ n whenever m divides n, where M is
(a) {2, 3, 4, 8}

(b) the set of all prime numbers.


Solution. .
(a) The maximal elements of M = {2, 3, 4, 8} are 3 and 8. Indeed, 3 does not divide any of the other
elements and 2 and 4 both divide 8, wheras 8 does not divide any of the other elements.

(b) Every element of M = {p | p prime} is maximal, since no prime number divides any other prime number.

1
Problem 2 (Problem 8, Chapter 4.2, p. 218). If a subadditive functional defined on a normed space X is
nonnegative outside of a sphere {x | kxk = r}, show that it is nonnegative for all x ∈ X.
Solution. Let p be a subadditive functional on X such that p(x) is nonnegative for all x ∈ X with kxk > r.
Note that p(y) = p(y + 0) ≤ p(y) + p(0) for all y ∈ X, and thus

0 = p(y) − p(y) ≤ p(0).

Hence 0 ≤ p(0) and thus p(0) is nonnegative. Let x ∈ X such that x 6= 0. If kxk > r then p(x) is nonnegative
r
by assumption, so suppose kxk ≤ r. Then there is an n ∈ N such that n > kxk and thus knxk > r. Then

p(x + · · · + x) ≤ p(x) + · · · + p(x) = np(x),


| {z } | {z }
n times n times

1
and thus n p(nx) ≤ p(x). But p(nx) ≥ 0, so p(x) ≥ 0 as well.

2
Problem 3 (Problem 8, Chapter 4.3, p. 224). Let X be a nomed space and X 0 its dual space. If X 6= {0},
show that X 0 cannot be {0}.
Solution. Let x ∈ X such that x 6= 0. By the Hahn-Banach Theorem, there exists a functional f ∈ X 0 such
that f (x) = kxk and kf k = 1. Hence X 0 6= {0}.

3
Problem 4 (Problem 8, Chapter 4.5, p. 238). Let X and Y be normed spaces and T ∈ B(X, Y ) such that
−1 ×
T −1 ∈ B(Y, X) exists. Show that (T × ) = T −1 .
Solution. First note that (1X )× = 1X 0 for any normed space X, where 1X is the identity mapping on X.
Indeed, for any f ∈ X 0 and x ∈ X we have

(1X )× f (x) = f (1X x) = f (x)




and thus (1X )× f = f for all X 0 .


Recall that, for operators S and T , we have (ST )× = T × S × . Take S = T −1 , then

1X 0 = (1X )× = (T −1 T )× = T × (T −1 )× ,

so (T −1 )× is the right-inverse of T × . Similarly,

1Y 0 = (1Y )× = (T T −1 )× = (T −1 )× T × ,
−1 ×
so (T −1 )× is also the left-inverse of T × . Hence, T × is invertible and (T × ) = T −1 .

4
Problem 5 (Problem 10, Chapter 4.5, p. 239 – Annihilator). Let B be a subset of the dual space X 0 of
a nomed space X. The annihilator a B of B is defined to be
a
B = {x ∈ X | f (x) = 0 for all f ∈ B} .

Let T : X −→ Y be a bounded linear operator. Show that R(T ) ⊂ a N (T × ). What does this mean with
respect to the task of solving an equation T x = y?
Solution. Let f ∈ N (T × ), then T × f = 0. That is,

(T × f )(x) = f (T x) = 0 for all x ∈ X.

Since this holds for all f ∈ N (T × ), we have T x ∈ a N (T × ) for all x ∈ X and thus R(T ) ⊂ a N (T × ).

This means that if there exists a functional f ∈ N (T × ) such that f (y) 6= 0, then y 6∈ a N (T × ) and thus y
is not in the range of T . Hence T x = y has no solution. This is equivalent to:

if f (y) 6= 0 for some functional f ∈ Y 0


such that f (T x) = 0 for all x ∈ X,
then T x = y has no solution.

5
Problem 6 (Problem 4, Chapter 4.6, p. 246). Show that a Banach space X is reflexive if and only if its
dual space X 0 is reflexive. (Hint: Show that a closed subspace of a reflexive Banach space is reflexive.)
Solution.
Lemma 1. Any closed subspace of a reflexive Banach space is reflexive.

Proof. Let X be a reflexive Banach space and Y ⊂ X be a closed subset. Let ϕ ∈ Y 00 and define ϕ̃ ∈ X 00 by

ϕ̃(f ) = ϕ(f |Y ) for all f ∈ X 0 .

Since X is reflexive, ϕ̃ is of the form ϕ̃ = ψx for some x ∈ X where

ψx (f ) = f (x) for all f ∈ X 0 .

So ϕ(f |Y ) = f (x). We claim that x ∈ Y . Indeed, otherwise x 6∈ Y and there exists a bounded linear
functional on X such that f |Y = 0 and f (x) 6= 0 (see Lemma 4.6-7 in Kreyszig). But this is a contradiction
to the fact that
f (x) = ϕ(f |Y ) = ϕ(0) = 0,
since f |Y = 0. By the Hahn-Banach Theorem, every linear functional g on Y ⊂ X can be written as g = f |Y
for some f ∈ X 0 , and thus

ϕ(g) = ϕ(f |Y ) = ϕ̃(f ) = hx (f ) = f (x) = f |Y (x) = g(x)

where x ∈ Y . Hence for each ϕ ∈ Y 00 there is an x ∈ Y such that ϕ(g) = g(x), and thus Y is reflexive.
Proposition 2. A Banach space X is reflexive if and only if its dual space X 0 is reflexive.
Proof. Assume that X is reflexive and let C1 : X −→ X 00 be the canonical isometry. Consider X 000 = (X 00 )0
and let C2 : X 0 −→ X 000 be the canonical embedding that maps f ∈ X 0 to functional ξf ∈ X 000 such that
ξf (ϕ) = ϕ(f ) for all ϕ ∈ X 00 . Let ξ ∈ X 000 and define a functional fξ ∈ X 0 by

fξ (x) = ξ(C1 (x)).

Since X is reflexive, for each ϕ ∈ X 00 there is an x ∈ X such that ϕ = C1 (x). Hence

(C2 (fξ ))(ϕ) = ϕ(fξ ) = fξ (x) = ξ(C1 (x)) = ξ(ϕ)

and thus ξ = C2 (fξ ). So the embedding C2 is surjective and is therefore an isometry, so X 0 is reflexive.
Now suppose that X 0 is reflexive. By the argument above, we have that X 00 is reflexive as well. Consider
R(C1 ) ⊂ X 00 , which is isometric to X. Since R(C1 ) is closed in X 00 , by the lemma above we have that R(C1 )
and thus X are also reflexive.

6
Problem 7 (Problem 6, Chapter 4.6, p. 246). Show that different closed subspaces Y1 and Y2 of a normed
space X have different annihilators.
Solution. Proof. Let Y1 , Y2 ⊂ X be two closed subspaces of X with Y1 6= Y2 . Without loss of generality, we
may assume that Y1 r Y2 6= ∅, then let y ∈ Y1 r Y2 . Since y 6∈ Y2 , by the Hahn-Banach Theorem (see Lemma
4.6-7) there exists a functional f ∈ X 0 such that f (y) 6= 0 and f |Y2 = 0. Thus f ∈ (Y2 )a but f 6∈ (Y1 )a ,
hence (Y1 )a 6= (Y2 )a .

7
Problem 8 (Problem 8, Chapter 4.6, p. 246). Let M be any subset of a normed space X. Show that an
x0 ∈ X is an element of A = span M if and only if f (x0 ) = 0 for every f ∈ X 0 such that f |M = 0.
Solution. Using the ‘annihilator’ notation, this is equivalent to saying that span M = a (Ma ).
Proof. Now suppose that x0 6∈ A = span M . By the Hahn-Banach Theorem (see Lemma 4.6-7), there is
a linear functional f ∈ X 0 such that f (x0 ) 6= 0 and f |A = 0. But M is a subset of A, so f |M = 0 with
f (x0 ) 6= 0 and thus x0 6∈ a (Ma ).

8
Problem 9 (Problem 4, Chapter 4.7, p. 254). Find a meager dense subset in R2 .
Solution. Note that Q is meager and dense in R. Futhermore, Q is countable and we may enumerate it as
Q = {q1 , q2 , q3 , . . . }. Consider the subset M ⊂ R2 defined by

[
M= Mi , where Mi = {(qi , x) | x ∈ R} .
i=1

That is, M consists of the union of parallel lines intersecting the x-axis at each rational point. Then M is
clearly dense in R2 , and each Mi is rare is R2 . Since M is a countable union of rare sets, it is meager in R2 .

9
Problem 10 (Problem 6, Chapter 4.7, p. 255). Show that the complement M c of a meager subset M of a
complete metric space X is nonmeager.
Solution. If X = ∅ then this is true vacuously. Let X 6= ∅ be a complete metric space and let M ⊂ X be a
meager subset. Note that we may write X as a union

X = M ∪ M c.

If M c were meager, then it may be written as a countable union of rare sets. Hence X is a countable union
of rare sets (since it is a union of two sets that are countable unions of rare sets), and thus X is meager
in itself. But this is a contradiction to Baire’s Category Theorem, which states that a nonempty complete
normed space must be non-meager in itself.

10
Problem 11 (Problem 8, Chapter 4.7, p. 255). Show that completeness of X is essential in the statement
of the Uniform Boundedness Theorem. (Hint: Consider X ⊂ `∞ consisting of all finite sequences x = {xj },
that is, xj nonzero for only finitely many j, and consider the operator Tn x = fn x = nxn .)
Solution. Recall the statement of the theorem.

Theorem 3 (Uniform Boundedness Theorem). Let {Tn } be a sequence of bounded linear operators Tn :
X −→ Y from a Banach space X into a normed space Y such that {kTn xk} is bounded for every x ∈ X.
That is, for every x there exists a cx such that kTn xk ≤ cx for all n. Then the sequence of the norms kTn k
is bounded.
As in the problem statement, let X ⊂ `∞ be the subspace consisting of finite sequences. Then X is not
complete. For each n let Tn = fn be the linear functional defined by fn (x) = nxn . Let x ∈ X, then x = {xj }
and there exists an N ∈ N such that xj = 0 for all j > N . Then

kTn xk = |fn (x)| = |nxn | = n|xn | ≤ n kxk

where cx = kxk depends on the choice of x, and thus {kTn xk} is bounded for each x ∈ X. However, kTn k = n
for each n. Indeed, from the above analysis we see that kTn k ≤ n, and choosing x = {1, 1, . . . , 1, 0, 0, · · · }
| {z }
n
with kxk = 1 we have kTn xk = n so kTn k ≥ n for each n. Thus {kTn k} is not bounded.

11
Problem 12 (Problem 12, Chapter 4.7, p. 255). Let X and Y be Banach spaces and Tn ∈ B(X, Y ) such
that {Tn x} is Cauchy in Y for every x ∈ X. Show that Tn x −→ T x for some T ∈ B(X, Y ).
Solution. Since every Cauchy sequence is bounded, we have that {kTn xk} is bounded for all x ∈ X and
thus {kTn k} is bounded by the Uniform Boundedness Theorem. Thus sup kTn k is finite.
n∈N

Since Y is complete and the sequence {Tn x} is Cauchy in Y for each x ∈ X, each {Tn x} converges to an
element in Y . Define a mapping T : X −→ Y by

T x = lim Tn x.
n→∞

This is clearly linear, since by the linearity of Tn we have

T (ax + by) = lim Tn (ax + by) = lim (aTn x + bTn y) = aT x + bT y


n→∞ n→∞

for all x, y ∈ X and a, b in the field F. This operator is also bounded, since for all x ∈ X we have

kT xk = lim Tn x = lim kTn xk

n→∞ n→∞
≤ sup kTn k kxk ,
n∈N

kT xk
where we use the fact that the norm is continuous, so it commutes with limits. Hence kxk ≤ sup kTn k < ∞
n∈N
for all x 6= 0, but this is finite and thus kT k ≤ sup kTn k < ∞, so T is also bounded and thus T ∈ B(X, Y )
n∈N
as desired.

12
Problem 13 (Problem 14, Chapter 4.7, p. 255). If X and Y are Banach spaces and Tn ∈ B(X, Y ) for
each n, show that the following statements are equivalent:
(a) {kTn k} is bounded,
(b) {kTn xk} is bounded for all x ∈ X,

(c) {|g(Tn x)|} is bounded for all x ∈ X and g ∈ Y 0 .


Solution. We first prove the following lemma.
Lemma 4. If {xn } is a sequence in a Banach space X such that {f (xn )} is bounded for each functional
f ∈ Y 0 , then {kxn k} is bounded.

Proof. For each f ∈ X 0 , define the sequence {ϕn } in X 00 by ϕn (f ) = f (xn ). Then the sequence {ϕn (f )}
is bounded for each f ∈ X 0 by assumption, and hence {kϕn (f )k} is bounded for each f . By the Uniform
Boundedness Theorem, we have that {kϕn k} is also bounded, but kϕn k = kxn k for each n and thus {kxn k}
is bounded.

We now proceed to prove the problem statement.


• (a) ⇒ (b). Since {kTn k} is bounded, there is a constant c ≥ 0 such that kTn k ≤ c for all n. Define the
constant cx = c kxk for each x ∈ X. Then

kTn xk ≤ kTn k kxk ≤ cx ,

and thus {kTn xk} is bounded for each x ∈ X.


• (b) ⇒ (a). This follows from the Uniform Boundedness Theorem.
• (a) ⇒ (c). Let g ∈ Y 0 and define the functionals fn ∈ X 0 for each n by fn (x) = g(Tn x). Since {kTn k}
is bounded, there is a constant c such that kTn k ≤ c for all n. Then we have

|fn (x)| = |g(Tn x)| ≤ kgk kTn k kxk


≤ kgk c kxk

for each x ∈ X and thus kfn k ≤ c kgk, so the sequence {kfn k} is bounded. From the (a) ⇒ (b) part of
this problem, this implies that {|fn (x)|} = {|g(Tn x)|} is bounded for all x.
• (c) ⇒ (b). Let x ∈ X and define the sequence {yn } in Y by yn = Tn x. Then the sequence {g(yn )}
is bounded for each g ∈ Y 0 . Since Y is a Banach space, Lemma 4 from above says that {kyn k} is
bounded, and thus {kTn xk} is bounded.

13
Problem 14 (Problem 2, Chapter 4.8, p. 262). Let X and Y be normed spaces, T ∈ B(X, Y ) and {xn } a
w w
sequence in X. If xn −→ x, show that T xn −→ T x.
Solution. Let g ∈ Y 0 and define a functional f ∈ X 0 by f (x) = g(T x). Then g(T xn ) = f (xn ) and thus
w
g(T xn ) −→ g(T x) if and only if f (xn ) −→ f (x), which is true since xn −→ x. Hence g(T xn ) −→ g(T x) for
w
all g ∈ Y 0 and thus T xn −→ T x.

14
w
Problem 15 (Problem 4, Chapter 4.8, p. 262). Show that xn −→ x implies lim inf kxn k ≥ kxk.
n→∞

Solution. By the Hahn-Banach theorem, there exists a functional f ∈ X 0 such that kf k = 1 and f (x) = kxk.
Since xn weakly converges to x, we have that f (xn ) −→ f (x), hence |f (xn )| −→ |f (x)| = kxk. Thus

kxk = |f (x)| = lim |f (xn )|


n→∞
= lim inf |f (xn )|
n→∞
≤ lim inf kf k kxn k
n→∞ |{z}
=1
= lim inf kxn k ,
n→∞

as desired.

15
Problem 16 (Problem 6, Chapter 4.8, p. 262). If {xn } is a weakly convergent sequence in a normed space
w
X, say xn −→ x, show that there is a sequence {ym } of linear combinations of elements of {xn } which
converges strongly to x.
Solution. We first prove the following lemma (see problem 4.8.1 in Kreeyszig).
w
Lemma 5. If xn −→ x in a normed space X, then x ∈ Y where Y = span{xn }.
Proof. Suppose otherwise that x 6∈ Y . Then the distance

δ = inf kx − yk
y∈Y

from x to Y is positive. By the theorem in class (Theorem 4.6-7 in Kreyszig), there exists a functional
f ∈ X 0 such that kf k = 1, f (x) = δ and f (y) = 0 for all y ∈ Y . Hence f (xn ) = 0 for each n, and thus f (xn )
w
does not converge to f (x) = δ, a contradiction to xn −→ x.

The proof of the problem statement follows trivially from this lemma. Indeed, since x ∈ Y , there is a
sequence {ym } in Y such that ym −→ x. But Y = span{xn }, and thus each ym is a linear combination of
elements in {xn }.

16
Problem 17 (Problem 8, Chapter 4.8, p. 262 – Weak Cauchy sequence). A weak Cauchy sequence in a
real or complex normed space X is a sequence {xn } in X such that for every f ∈ X 0 the sequence {f (xn )}
is Cauchy in R or C, respectively. Show that every weak Cauchy sequence is bounded.
Solution. For clarity, denote F = R or C.
Define the sequence {ϕn } in X 00 such that ϕn (f ) = f (xn ) for all f ∈ X 0 . Let f ∈ X 0 be arbitrary. Since
{xn } is weak Cauchy, we have that {f (xn )} is Cauchy in F and thus the sequence {|f (xn )|} is bounded.
Hence {|ϕn (f )|} is bounded for each f ∈ X 0 . But X 0 is Banach, so we have that {kϕn k} is bounded by the
Uniform Boundedness Theorem. Then {xn } is bounded since kxn k = kϕn k.

17
Problem 18 (Problem 10, Chapter 4.8, p. 263 – Weak completeness). A normed space X is said to be
weakly complete if each Cauchy sequence in X converges weakly in X. If X is reflexive, show that X is
weakly complete.
Solution. Let {xn } be a Cauchy sequence in X. For each xn ∈ X let ϕn ∈ X 00 be the functional defined
by ϕn (f ) = f (xn ) for all functionals f ∈ X 0 . Since X is reflexive, we have X ∼
= X 00 so {ϕn } is a Cauchy
00
sequence in X . Let ε > 0, then there exists an N ∈ N such that kϕn − ϕm k ≤ kfε k for all n, m ≥ N since
{ϕn } is Cauchy. Then for each n, m ≥ N we have

|f (xn ) − f (xm )| = |ϕn (f ) − ϕm (f )|


≤ kϕn − ϕm k kf k
< ε,

so the sequence {f (xn )} is Cauchy in R and thus {xn } is weakly convergent. Then X is weakly complete,
since every Cauchy sequence in X is weakly convergent.

18

You might also like